LSAT and Law School Admissions Forum

Get expert LSAT preparation and law school admissions advice from PowerScore Test Preparation.

User avatar
 Dave Killoran
PowerScore Staff
  • PowerScore Staff
  • Posts: 5853
  • Joined: Mar 25, 2011
|
#94750
Complete Question Explanation
(The complete setup for this game can be found here: lsat/viewtopic.php?f=269&t=1940)

The correct answer choice is (D)

From the third rule, M does not stock aisle 1, and so answer choice (A) can be eliminated.

From the sequence created by the combination of the fifth and seventh rules, M cannot stock aisle 9, and so answer choice (B) can be eliminated.

The difference between answer choices (C), (D), and (E) is that (C) and (E) are incomplete lists of the aisles M could stock. As the question stem asks for a “complete list,” and answer choice (D) contains the complete list, it is the correct answer.

Get the most out of your LSAT Prep Plus subscription.

Analyze and track your performance with our Testing and Analytics Package.